Le 4

You might also like

Download as pdf or txt
Download as pdf or txt
You are on page 1of 35

LEGALEDGE TEST SERIES

“Part of the most Comprehensive Classroom Training, Prep Content & Test Series across the Nation.
From the producers of the Best and the most number of A.I.Rs in the past years, every year!”

MOCK COMMON LAW ADMISSION TEST 2021

MOCK CLAT #04


Candidate Name : _________________
Duration : 120 Minutes
Batch : _________________
Max. Marks : 150
Contact No. : _________________
Centre Name : __________
Date of Exam : _________________

INSTRUCTIONS TO CANDIDATES

1. No clarification on the question paper can be sought. Answer the questions as they are.
2. There are 150multiple choice objective type questions.
3. Each question carries ONE mark. Total marks are 150.
4. There is a negative marking of 0.25 marks for every incorrect answer.
5. Candidates have to indicate the correct answer by darkening one of the four responses provided,
with a BALL PEN (BLUE OR BLACK) in the OMR Answer Sheet.
Example: For the question, "Where is the TajMahal located?", the correct answer is (b).
The candidate has to darken the corresponding circle as indicated below :
(a) Kolkata (b) Agra (c) Bhopal (d) Delhi
Right Method Wrong Methods

6. Answering the questions by any method other than the method indicated above shall be considered
incorrect and no marks will be awarded for the same.
7. More than one response to a question shall be counted as wrong.
8. The candidate shall not write anything on the OMR Answer Sheet other than the details required
and in the spaces provided for.
9. After the Test is over, the candidate has to return the OMR Answer Sheet to the invigilator. The
T49_MC15_ALL_03_019-20

candidate should take the Test Paper along with them.


10. The use of any unfair means by any candidate shall result in the cancellation of his/her candidature.
11. Impersonation is an offence and the candidate, apart from disqualification, may have to face
criminal prosecution.
12. Electronic gadgets like mobile phones, pagers or calculators are strictly not permitted inside the
Test Centre/Hall.
13. The candidates shall not leave the hall before the Test is over.
MOCK CLAT #04

SECTION-A : ENGLISH LANGUAGE

Directions (Q.1-Q.29): Read the passages carefully and answer the questions based on it.

Passage (Q.1-Q.5): In traditional theater forms, the roles of performer and audience are completely
separate, so that performance space can be said to encompass an actors‘ sphere and a spectators‘
sphere. Even when performers move out into the audience or when there is scripted audience
interaction, spectators do not become performers. Finally, while stories may open up the imagination or
excite audiences, according to Augusto Boal, they discourage political action by providing catharsis. The
passive spectator follows the play‘s emotional arc and, once the action concludes, finds the issue closed.
Boal reminds us that our theater etiquette creates a kind of culture of apathy where individuals do not act
communally, despite shared space, and remain distanced from art.

Workshop theater, such as Boal‘s Image Theatre and Forum Theatre, is a response to that. In the
workshop form, performance space is created for a select group of people, but the performers‘ sphere
and the audience‘s sphere are collapsed: everyone is at once theater maker and witness. In Image
Theatre, participants will come up with a theme or issue and arrange themselves into a tableau that
depicts what that issue looks like in society today, versus what the ideal situation would be. They then try
to transition from the current image to the ideal image in a way that seems plausible to all the
participants. Forum Theatre, on the other hand, creates a narrative skit depicting a certain problem. After
the actors have gone through the action of the play once, a facilitator, known as the joker (like the one in
a pack of cards), encourages those who have watched the story to watch it again and to stop it at any
time to take the place of the protagonist. The aim is to find a solution to the problem, realizing along the
way all of the obstacles involved. In Forum Theatre, just as in Image Theatre, there is not always a
solution. The main goal of this form, then, is to engage in the action, to reflect, and to understand
particular issues as being part of a larger picture, thus using art to re-cast what seem like private troubles
in a public, political light.

The main reason Boal developed these workshop styles was to grant audiences agency so that they may
create ways to free themselves of oppression. Because he found theater audiences to be locked into a
passive role—just like he found the oppressed coerced into a subservient role in relation to their
oppressors—he created the ―spectator,‖ or someone who simultaneously witnesses and creates theater.

1. The second paragraph of the passage serves to


(a) elaborate on the topic of the first paragraph
(b) provide a rationale for an artistic endeavor
(c) discuss an artistic answer to a passive culture
(d) explain the theater‘s lack of appeal

2. The author uses the word agency to mean


(a) profit (b) organization (c) publicity (d) power

3. Which of the following would Boal consider a ―spectator‖?


(a) a person who engages in political action
(b) an audience member who finds catharsis in a play
(c) any person placed in a subservient role
(d) a participant in an Image workshop

4. According to Boal, all of the following are disadvantage of traditional theater forms EXCEPT:
(a) Such productions prevent the actors from going into the audience.
(b) Such productions provide catharsis.
(c) Such productions discourage communal activity.
(d) Such productions obstruct political change.

Head Office: 127, Zone II, MP Nagar, Bhopal |+91-9111555433|www.legaledge.in Page 2 of 35


MOCK CLAT #04

5. All of the following would be characteristic of a Forum workshop EXCEPT:


(a) Productions begin with a narrative script.
(b) Different people often play the protagonist.
(c) Some performances do not achieve catharsis.
(d) Participants arrange themselves into a tableau.

Passage (Q.6-Q.9): It is one of the disadvantages of reading books about natural scenic wonders that
they fill the mind with pictures, often exaggerated, often distorted, often blurred, and, even when well
drawn,injurious to the freshness of first impressions.
Such has been the fate of us with regard to the Falls of Niagara. There was little accuracy in the
estimates of The first obseversservers of the cataract. Startled by an exhibition of power of so novel and
so grand, emotion leaped beyond the control of the judgementand gave currency to notions which have
often led to disappointment.

In the winter of 1678 and 1679 the cataract was visited by father Hennepin, and described in a book
dedicated to the king of Great Britain. He gives a drawing of the waterfall,
which shows that serious changes have taken place since his time. He describes it as 'a great and
prodigious cadence
ofwater, to which the universe does not offer a parallel'.the height of fall, according to Hennepin, was
more than 600 feet. The waters', he says, 'which fall from this great precipice do foam and boil in the
most astonishing manner, making a noise more terrible than that of thunder. When the wind blows to the
south its frightful roaring may be heard for more than fifteen leagues'. The Baron la Hontan, who visited
Niagara in 1687, makes the height 800 feet. In 1721 Charlevois, in a letter to Madame de Maintenon,
after referring to the exaggerations of his predecessors, thus states the result of his own observations:
'For my part, after examining it on all sides, I am inclined to think that we cannot allow it less than 140 or
150 feet', — a remarkably close estimate.
As regards the noise of the fall, Charlevois declares the accounts of his predecessors, which, I may say,
are repeated to the present hour, to be altogether extravagant. He is perfectly right. The thunders of
Niagara are formidable enough to those who really seek them at the base of the Horseshoe Fall; but on
the banks of the river, and particularly above the fall, its silence, rather than its noise, is surprising. 'his
arises, in part, from the lack of resonance; the surrounding country being flat, and therefore furnishing no
echoing surfaces to reinforce the shock of the water.

6. The author of the passage is primarily concerned with doing which of the following?
(a) Discussing a problem associated with reading books about natural wonders before visiting them.
(b) Refuting those who claimed that a natural wonder was of a particular height.
(c) Describing how the initial descriptions of a natural wonder were greatly exaggerated.
(d) Explaining why an expected aural phenomenon does not actually occur.

7. Which of the following statements can be inferred from the passage about the height of the Niagara
Falls?
(a) The Niagara Falls were higher than any other natural waterfall at the time of writing the passage.
(b) Hennepin lied about the height of the Niagara Falls in his book dedicated to the King of Great Britain.
(c) There are no mountains in the area surrounding the Niagara Falls.
(d) The actual height of the Niagara Falls is around 150 feet.

8. According to the information in the passage, each of the following is true EXCEPT:
(a) The Niagara Falls does make a thunderous sound but it is not audible from everywhere.
(b) Reading books about natural scenery can be disadvantageous at times.
(c) A flat topography is not conducive to resonance.
(d) A person standing at the base of the Niagara Falls would actually be surprised at the silence that
greets him.

Head Office: 127, Zone II, MP Nagar, Bhopal |+91-9111555433|www.legaledge.in Page 3 of 35


MOCK CLAT #04

9. If after reading this passage a person were to visit the Niagara Falls, which of the following would he NOT
be surprised at?
(a) The Niagara Falls is 480 feet in height.
(b) There are several tall mountains in the vicinity of the Niagara Falls.
(c) There is absolute silence at the base of the Niagara Falls.
(d) There is a river flowing in the vicinity of the Niagara Falls.

Passage (Q.10-Q.15): If you talk about the history of mathematics to any Indian mathematician, you are
most likely to have two types of reactions. Firstly, his mind would immediately flash back to the great
works of ancient Indian mathematicians, including the mathematical contents of the Vedas. Secondly, he
remonstrates that he does not know the history of mathematics in detail, nor is he interested in learning it;
in fact, he has no time to learn it. However, the history of mathematics cannot be separated from
mathematics. As Andre Weil put it, the history of mathematics in itself is mathematics and no one should
venture to enter the field unless he knows enough of mathematics. Placed between these two
extraordinary, adventitious and redundant situations, the studies of history of mathematics have suffered
hopelessly. In fact, they have yet to be initiated in the right perspective. While a good university in every
part of the world has a department of study in the history and philosophy of mathematics, it is
disappointing that there is hardly a university in India that provides facilities for such studies. In fact, a
large number of Indian manuscripts are yet to be translated from the original languages in which they
detail facts about ancient mathematics.
This has led to a deadening disposition. Due to the vacuum of studies on the history of mathematics in
India, studies by Western historians and Euro-centric academics of history of mathematics have taken
centre stage in our minds and syllabi. These historians and academicians deliberately ignore the
mathematical achievements of India. Their belief that except for the discovery of the concept of zero and
the decimal representation of numbers, everything else in mathematics was done outside India is now
universally accepted. The reason for such thinking is easy to understand. The whole of Europe learned
mathematics through the Greeks. The net result of such historians is that the history of Indian
mathematics is not even considered as a part of the world history of mathematics. For how long can we
continue to believe in the half-truths propagated by the Western historians and Euro-centric academics?
Keeping this in mind, the authors should be commended for endeavouring to take up this prodigious task.
Their aim was to highlight momentous, positive and concrete contributions made by Indian
mathematicians in the initial advancement of mathematics and relate them to the developments that
occurred in a later era in Greece, Middle East, China and Japan.

10. The Passage is most likely to be


(a) A newspaper article lamenting the lack of scientific temper among Indian historians.
(b) A blog essaying the history of mathematics from ancient times to the modern era.
(c) A preface or foreword in a book written to explore contributions made by Indian mathematicians in the
initial advancement of mathematics.
(d) An article in a journal of mathematics describing the contributions made by Indian mathematicians.

11. Match the following words used in the Passage with their meanings.

Words Meanings
Remarkably or impressively great in extent, size,
i Remonstrate A
or degree
ii Adventitious B Of great importance or significance
Happening as a result of an external factor or
iii Prodigious C
chance rather than design or inherent nature
iv Momentous D To reason or plead in protest
(a) i - B, ii - C, iii - A, iv - D (b) i - C, ii - B, iii - D, iv - A
(c) i - D, ii - C, iii - B, iv - A (d) i - D, ii - C, iii - A, iv - B

Head Office: 127, Zone II, MP Nagar, Bhopal |+91-9111555433|www.legaledge.in Page 4 of 35


MOCK CLAT #04

12. Which of the following would be the most appropriate title for the Passage?
(a) Eurasia: The Cradle of History of Mathematics
(b) India and Greece: Civilizations United by Mathematics, Divided by Destiny
(c) Academic Acrobatics: Negation of Contribution by Indians in Initial Advancement of Mathematics
(d) Mathematical Contributions: A Perspective by Western historians and Euro-centric Academics

13. Based on the Passage, which of the following can be said to be true?
(a) Historians of Indian origin have a tendency to exaggerate claims of historic contribution of Indian
mathematics and Indian universities do not have the wherewithal to fund courses on history of Indian
mathematics.
(b) The belief of Euro-centric academics that except for the discovery of the concept of zero and the
decimal representation of numbers, everything else in mathematics was done outside India is true.
(c) As some texts of Indian history cannot be translated from their original languages, the contribution
made by Indian mathematicians in the initial advancement of mathematics may not be fully known.
(d) The history of Indian mathematics is not even considered as a part of the world history of
mathematics due to the Euro-centric academics that have spread a motivated version of the history of
mathematics and the lack of multiple studies determining positive and concrete contributions made by
Indian mathematicians in the initial advancement of mathematics.

14. Which of the following messages can be inferred from the Passage?
i. Indian mathematicians do not have the time, energy, resources or orientation to study about the
history of mathematics and the contribution of Indian mathematics to the history of mathematics.
ii. Highlighting concrete contributions made by Indian mathematicians in the initial advancement of
mathematics and relating them to the developments that occurred in a later era in Greece, Middle
East, China and Japan will expose the deception spread by the Euro-centric academicians about their
contribution.
iii. Unless a nation and its people work to own their place in history and present it in an appropriate
manner, they will be slaves of external projections of their histories.
iv. As history cannot be undone, it is important for academics to move ahead from the deification of the
contribution of Indians to the history of mathematics and work practically on finding new innovations
in fields such as mathematics.
v. As they have more credibility than Indian mathematicians, it is essential to find Western historians
and Euro-centric academics of history of mathematics that are willing to dig out the contribution of
Indian mathematics to the history of mathematics and fund their studies.
(a) Both (iii) and (v) (b) Both (ii) and (iii)
(c) Both (ii) and (iv) (d) All except (iii)

15. Which of the following words can replace the word 'deadening' as used in the Passage?
(a) Injuring (b) Destroying (c) Stunning (d) Dulling

Passage (Q.16-Q.22): Some observers have attributed the dramatic growth in temporary employment
that occurred in the United States during the 1980s to increased participation in the workforce by certain
groups, such as first-time or reentering workers, who supposedly prefer such arrangements. However,
statistical analyses reveal that demographic changes in the workforce did not correlate with variations in
the total number of temporary workers. Instead, these analyses suggest that factors affecting employers
account for the rise in temporary employment. One factor is product demand: temporary employment is
favored by employers who are adapting to fluctuating demand for products while at the same time
seeking to reduce overall labor costs. Another factor is labor‘s reduced bargaining strength, which allows
employers more control over the terms of employment. Given the analyses, which reveal that growth in
temporary employment now far exceeds the level explainable by recent workforce entry rates of groups
said to prefer temporary jobs, firms should be discouraged from creating excessive numbers of temporary
positions. Government policymakers should consider mandating benefit coverage for temporary
employees, promoting pay equity between temporary and permanent workers, assisting labor unions in
organizing temporary workers, and encouraging firms to assign temporary jobs primarily to employees
who explicitly indicate that preference.

Head Office: 127, Zone II, MP Nagar, Bhopal |+91-9111555433|www.legaledge.in Page 5 of 35


MOCK CLAT #04

16. The primary purpose of the passage is to


(a) present the results of statistical analyses and propose further studies
(b) explain a recent development and predict its eventual consequences
(c) identify the reasons for a trend and recommend measures to address it
(d) outline several theories about a phenomenon and advocate one of them

17. According to the passage, which of the following is true of the ―factors affecting employers‖ that are
mentioned in the passage?
(a) Most experts cite them as having initiated the growth in temporary employment that occurred during
the 1980s.
(b) They may account for the increase in the total number of temporary workers during the 1980s.
(c) They were less important than demographic change in accounting for the increase of temporary
employment during the 1980s.
(d) They included a sharp increase in the cost of labor during the 1980s.

18. The passage suggests which of the following about the use of temporary employment by firms during the
1980s?
(a) It enabled firms to deal with fluctuating product demand far more efficiently than they did before the
1980s.
(b) It increased as a result of increased participation in the workforce by certain demographic groups.
(c) It was discouraged by government-mandated policies.
(d) It increased partly as a result of workers‘ reduced ability to control the terms of their employment.

19. The passage suggests which of the following about the workers who took temporary jobs during the
1980s?
(a) Their jobs frequently led to permanent positions within firms.
(b) They constituted a less demographically diverse group than has been suggested.
(c) They were occasionally involved in actions organized by labor unions.
(d) They did not necessarily prefer temporary employment to permanent employment.

20. The first sentence in the passage suggests that the ―observers‖ mentioned in the passage would be most
likely to predict which of the following?
(a) That the number of new temporary positions would decline as fewer workers who preferred temporary
employment entered the workforce
(b) That the total number of temporary positions would increase as fewer workers were able to find
permanent positions
(c) That employers would have less control over the terms of workers‘ employment as workers increased
their bargaining strength
(d) That more workers would be hired for temporary positions as product demand increased

21. In the context of the passage, the word ―excessive‖ most closely corresponds to which of the following
phrases?
(a) Far more than can be justified by worker preferences
(b) Far more than can be explained by fluctuations in product demand
(c) Far more than can be beneficial to the success of the firms themselves
(d) Far more than can be accounted for by an expanding national economy

22. The passage mentions each of the following as an appropriate kind of governmental action EXCEPT
(a) getting firms to offer temporary employment primarily to a certain group of people
(b) encouraging equitable pay for temporary and permanent employees
(c) facilitating the organization of temporary workers by labor unions
(d) establishing guidelines on the proportion of temporary workers that firms should employ

Head Office: 127, Zone II, MP Nagar, Bhopal |+91-9111555433|www.legaledge.in Page 6 of 35


MOCK CLAT #04

Passage (Q.23-Q.29): The following is adapted from a medical reference guide regarding the heart and
cardiovascular system.
The heart is responsible for moving blood to all of the body's tissues through a 60,000-mile network of
vessels. The pumping of the heart relies on an intricate system of muscle (myocardium),valves, coronary
vessels, the conduction (electrical) system, arteries and veins, and the sac around the heart
(pericardium) .

The human heart is divided into four chambers, the walls of which are made of the myocardium, the
muscle that contracts rhythmically under the stimulation of electrical currents. The myocardium is
composed of individual muscle cells called myocytes, which work together to contract and relax the heart
chambers in the correct sequence to pump blood to the lungs and the body. The heart is able to pump
blood in a coordinated manner because of the arrangement of the cells and the electrical messages that
pass easily between the cells. This cardiovascular pump operates by squeezing blood out of its
chambers (contraction) and then expanding to allow blood in (relaxation). The action is similar to
squeezing water out of a soft plastic bottle while holding it underwater and then releasing one's grasp so
that water is sucked back into the bottle as it re-expands.

The right side of the heart, which is composed of the right atrium and right ventricle, is responsible for
pulmonary circulation. That is, it pumps blood through the lungs, where it receives oxygen and rids itself
of carbon dioxide. The left side of the heart, composed of the left atrium and left ventricle, receives the
newly oxygenated blood and pumps it through the body where it delivers oxy-gen and picks up carbon
dioxide (waste). Blood must circle from the right side of the heart and through the lungs before being
delivered to the left side and throughout the body.

"Used blood" returns to the right side of the heart via two large veins-the superior vena cava (from the
head and arms) and the inferior venacava (from the legs and abdomen). Blood from the right heart is
dark bluish red because it is deoxygenated, or lacks oxygen. The blood from the left heart is oxygenated
and therefore is bright red. Blood from the left heart is delivered to the body through the aorta, the largest
blood vessel in the body.

Because the heart never rests while it supplies blood to the rest of the body, it actually works harder than
any other muscle in the body and needs a much richer blood supply than other muscles. Although the
heart makes up less than I percent of a person's body weight, it requires 4 to 5 percent of its blood.

23. The passage is primarily concerned with


(a) pulmonary circulation (b) the structure and function of the heart
(c) blood supply to the heart (d) contraction and relaxation of the heart

24. Myocytes are muscle cells responsible for


(a) opening and closing the heart valves (b) providing oxygen to the blood
(c) allowing the heart to rest (d) contracting and relaxing the heart chambers

25. "Relaxation" most nearly means


(a) resting (b) squeezing blood out of its chambers
(c) pumping (d) expanding to allow blood in
26. Pulmonary circulation is
(a) the right heart chamber
(b) the pumping of blood through the lungs, where it is oxygenated
(c) the pumping of blood through the aorta, which delivers it to the body
(d) the left heart chamber
27. "Oxygenated" most nearly means
(a) supplied with oxygen (b) in need of oxygen
(c) supplied with oxygen and carbon dioxide (d) bright red

Head Office: 127, Zone II, MP Nagar, Bhopal |+91-9111555433|www.legaledge.in Page 7 of 35


MOCK CLAT #04

28. The author's use of quotation marks around "usedblood" implies


(a) the blood is now waste
(b) the blood is bright red
(c) the blood is deoxygenated and is now waste
(d) the blood has circulated throughout the body

29. According to the passage, the heart requires how much of one's blood supply?
(a) Less than 1 % (b) The same amount of blood as the body'sother organs
(c) 4%-5% (d) A richer supply

Head Office: 127, Zone II, MP Nagar, Bhopal |+91-9111555433|www.legaledge.in Page 8 of 35


MOCK CLAT #04

SECTION-B : GENERAL KNOWLEDGE/CURRENT AFFAIRS

Directions (Q.30-Q.68): Read the information carefully and answer the questions based on it.

Passage (Q.30-Q.34): More popularly known by the title, ‗[1]‘, renowned conservationist JadevPayeng of
Assam was feted with the prestigious Karmayogi Award for 2020 in New Delhi .

Payeng received the honour in recognition of his contribution to afforestation on a sandbar of the River
Brahmaputra. Now named Molai after Payeng‘s nickname, the sandbar is part of [2], the world‘s largest
river island.

Speaking after his felicitation, Payeng told a full house, ―My work is above religion or politics; it is about
nature!‖

He then went on to speak about his journey in life and the dire need to make people aware of the need to
conserve nature, sprinkling his narration with interesting episodes from his own eventful life.

Once when he learned that some people had come to his area to fell down trees, he confronted them
with the warning, ―Cut me up before you cut them down. I won‘t allow you to cut down any trees!‖

Additional commissioner of Delhi Police and co-head of SPUNER, HibuTamang, who was also present
on the dais, said that achievers like Payeng were a source of inspiration for everyone, including people of
the Northeast region.

30. Fill [1] with a suitable option.


(a) Environmentalist of India
(b) Forest Man of India
(c) Climate Checker of India
(d) Zoologist of India

31. In 2020, JadevPayeng awarded the ______ edition of Karmayogi award, instituted by _______.
(a) 4th , Forest India
(b) 6th, My Home India
(c) 4th, My Home India
(d) 6th, Forest India

32. Which is the world‘s largest riverine island, redacted as [2] in the above passage?
(a) Munroe Island
(b) Majuli Island
(c) Divar Island
(d) Rica Island

33. JadavPayeng has been awarded with the fourth-highest civilian honour of India. What is the name of the
award?
(a) Bharat Ratna
(b) Padma Bhushan
(c) Padam Shri
(d) Padma Vibhushan

34. Which of the following are the United Nations highest environmental honour?
(a) Activist of Earth
(b) Champions of the Earth
(c) Creators of the Earth
(d) Makers of the Earth

Head Office: 127, Zone II, MP Nagar, Bhopal |+91-9111555433|www.legaledge.in Page 9 of 35


MOCK CLAT #04

Passage (Q.35-Q.38): The European Green Deal aims to transform the 27-country bloc from a high- to a
low-carbon economy, without reducing prosperity and while improving people‘s quality of life, through
cleaner air and water, better health and a thriving natural world. Climate neutrality, which can also be
expressed as a state of net-zero emissions, is realised when a country‘s emissions are balanced by
absorptions and removal of [1] from the atmosphere.

The massive green stimulus program, called [2], is aimed at giving a fillip to renewable energy sector,
accelerate the transition to clean mobility, and also increase energy savings. The funds will, however,
have to get the majority vote in the European Parliament and the unanimous nod of all the 27 members
of the bloc in the European Council, which may take months in negotiations. As the world grapples with
the coronavirus pandemic, governments across the world are under pressure to balance the transition
from managing a health crisis to charting out an economic recovery.

Source: Edited and recreated from the article ―What is the European Green Deal and will it really cost
€1tn?‖ from The Guardian

35. Which of the following components has been redacted by [1]?


(a) Carbon
(b) Chlorofluorocarbons
(c) Green House Gases
(d) Both (b) and (c)

36. The stimulus name has been redacted by [2]. What is it?
(a) EU Green Deal
(b) Next Generation EU
(c) Green Gen EU
(d) European Climate Pact

37. Who out of the following is the President of the European Commission?
(a) Charles Michel
(b) Christine Lagarde
(c) David Sassoli
(d) Ursula von der Leyen

38. The deal has set up a bloc-wide goal of net zero carbon emissions by the year -
(a) 2040
(b) 2045
(c) 2050
(d) 2055

Passage (Q.39-Q.43): The Indian Finance Minister NirmalaSitharaman announced [1] measures for the
MSME sector as part of the Modi government‘s ₹20 lakh crore stimulus package.The [1] measures for
the Micro, Small and Medium Enterprises (MSME) include collateral-free loans, additional debt and equity
infusion. The government has also announced a new policy that will shield the Indian MSME businesses
from unfair competition from foreign companies.
According to the MSME Development Act, [2], enterprises which are in the manufacturing and service
sectors qualify as MSMEs, subject to investment and turnover limits.
―MSMEs are critical for employment preservation and growth; firing up the MSME sector is the surest way
to accelerate job creation. The Finance Minister‘s schemes announced today will provide relief for
MSMEs that need immediate succor for their survival as well as help those enterprises that need a fillip to
grow. Expanding the definition of MSMEs and introducing new thresholds based on turnover while
elevating the investment ceilings for each segment are welcome." said, Arun M. Kumar, Chairman and
CEO, KPMG."Reduction of TDS being taken by the GOI will positively infuse the much-needed liquidity in
such turbulent times. Very bold and decisive steps by the Modi government." said, Rajat Prakash,
Partner, Athena Legal.

Head Office: 127, Zone II, MP Nagar, Bhopal |+91-9111555433|www.legaledge.in Page 10 of 35


MOCK CLAT #04

39. Fill [1] with a suitable option.


(a) 2
(b) 4
(c) 6
(d) 10

40. Fill [2] with a suitable option.


(a) 2000
(b) 2002
(c) 2006
(d) 2010

41. Recently, definition of MSME has changed by Finance minister. What is the reason behind it?
(a) To allow for relatively bigger companies to avail the benefits of cheaper and priority loans
(b) To include agriculture based industries
(c) To make eligible for WTO loans
(d) To promote Make in India

42. An emergency credit line of ₹3 lakh crore for MSMEs from Banks and NBFCs. The credit line will be up to
______ of all outstanding credit as on February 29, 2020.
(a) 10%
(b) 20%
(c) 30%
(d) 40%

43. Which of the following is not the measure taken by Finance Minister for MSME sector?
(a) ₹3 lakh crore collateral-free Automatic Loans for Businesses, including MSMEs
(b) ₹20,000 crore subordinate debt for Stressed MSMEs
(c) Global tenders to be allowed up to ₹200 crores
(d) All the pending dues from the government or government-owned companies will be cleared in the
next 45 days.

Passage (Q.44-Q.48): Issues concerning the production localisation of [1] light utility helicopters are in
the process of being resolved, N.M. Srikanth, Chief Executive Officer (CEO) of the Indo-Russian
Helicopters Limited (IRHL). The final deal for manufacturing the helicopters in India has been held up for
a while.
At the ongoing Defexpo 2020, Russian Helicopters (RH) signed a road map with IRHL for localisation of
[1] helicopter production in India.
Putin and Modi inspected a [1] that was flown to Vladivostok where the leaders had arrived for the
Eastern Economic Forum summit. Putin was quoted by TASS as telling Modi, ―I flew on this helicopter, I
liked it, it‘s really comfortable. It‘s good because it has coaxial rotors, there is no back rotor.
[1] is a design of Russia's famed Kamov design bureau. It is a light helicopter, with a maximum take-off
weight of over 3.5 tonnes and can carry a payload of up to 1 ton.
44. Which light utility helicopter can be replaced with [1] in the above passage?
(a) Kalashnikov
(b) S400
(c) Kamov 226 T
(d) Kazan Ansat
45. Which Indian company is part of joint-venture with Russian company to manufacture above said
helicopters?
(a) Hindustan Aeronautics Limited
(b) DRDO
(c) Rostech Limited
(d) None of the above
Head Office: 127, Zone II, MP Nagar, Bhopal |+91-9111555433|www.legaledge.in Page 11 of 35
MOCK CLAT #04

46. Why Coaxial rotors are suitable for India?


(a) They provide good thrust at higher altitudes like Himalayas
(b) They are maintenance free rotors
(c) This technology was first invented by India
(d) Their blades are of material that is only available in India

47. In which city 11th edition of DefExpo 2020 was organized?


(a) Delhi
(b) Bangalore
(c) Hyderabad
(d) Lucknow

48. Which indigenous air to ground missile was unveiled at DefExpo 2020?
(a) Khatarnak
(b) Khagantak
(c) Urja VII
(d) Urja VII

Passage (Q.49-Q.53): This year‘s Corruption Perceptions Index (CPI) reveals that a majority of countries
are showing little to no improvement in tackling corruption.
Our analysis also shows corruption is more pervasive in countries where big money can flow freely into
electoral campaigns and where governments listen only to the voices of wealthy .
The index ranks 180 countries and territories by their perceived levels of public sector corruption,
according to experts and business people. It uses a scale of zero to 100, where zero is highly corrupt and
100 is very clean. More than two-thirds of countries score below 50 on this year‘s CPI, with an average
score of just 43. Similar to previous years, the data shows that despite some progress, a majority of
countries are still failing to tackle public sector corruption effectively.
The bottom countries are Somalia, South Sudan and Syria with scores of 9, 12 and 13, respectively.
These countries are closely followed by Yemen (15), Venezuela (16), Sudan (16), Equatorial Guinea (16)
and Afghanistan (16).

49. India‘s ranking in the Corruption Perceptions Index (CPI-2019) has slipped from ___ to _____ compared
to the year 2018.
(a) 69,72
(b) 72,80
(c) 78,80
(d) 80,85

50. Which of the following Global Agency has released the corruption perception index, 2019 ?
(a) Greenpeace
(b) Freedom House
(c) Transparency International
(d) Reporters without Borders
51. When was the index first published by the above organization?
(a) 1888
(b) 1995
(c) 2000
(d) 2016
52. Which of the following countries topped the Index?
(a) Denmark
(b) USA
(c) UK
(d) Sweden

Head Office: 127, Zone II, MP Nagar, Bhopal |+91-9111555433|www.legaledge.in Page 12 of 35


MOCK CLAT #04

53. Transparency International has recommended a series of measures to combat rising corruption across
the world. Which of the following is/are not one of them?
(a) Manage conflicts of interest
(b) Control political financing
(c) Support preferential treatment
(d) Reinforce checks and balances

Passage (Q.54-Q.58): Black carbon concentrations near the Gangotri glacier rose 400 times in summer
due to [1] and [2], and triggered glacial melt, says a study by scientists at the Wadia Institute of
Himalayan Geology (WIHG).

The team of scientists from WIHG, led by P.S. Negi, measured variations of black carbon concentration
at Chirbasa, near the Gangotri glacier , located at an altitude of 3,600 metres, during the year 2016. ―The
monthly mean concentration of EBC (equivalent black carbon) was found to be minimum in August and
maximum in the month of May. The observed seasonal mean concentrations of EBC indicated a pristine
glacial source and an absence of EBC sources in the locality,‖ a press statement noted.

The concentration varied from a minimum of 0.01g/cubic metre in winter to 4.62g/cubic metre during
summer.

54. Fill [1] and [2] with a suitable option.


(a) Vehicular pollution and defence equipment testing
(b) Vehicular pollution and stubble burning
(c) Stubble burning and forest fires
(d) Forest fires and Vehicular pollution

55. What is the lifespan of Black Carbon in the atmosphere?


(a) 20 years
(b) 10 years
(c) 2 years
(d) Some days to weeks

56. India is the _______ emitter of black carbon in the world.


(a) Largest
(b) Second largest
(c) Least
(d) Third largest

57. Gangotri Glacier is located in which state of India?


(a) Himachal Pradesh
(b) Uttarakhand
(c) Jammu and Kashmir
(d) Punjab

58. How Black Carbon is originated in the atmosphere?


(a) By incomplete combustion of fossil fuel and biomass
(b) By complete combustion of fossil fuel and biomass
(c) By the use of air conditioning devices
(d) By the acid rain

Head Office: 127, Zone II, MP Nagar, Bhopal |+91-9111555433|www.legaledge.in Page 13 of 35


MOCK CLAT #04

Passage (Q.59-Q.63): Prime Minister NarendraModi participated in the online Summit of Non Aligned
Movement (NAM) Contact Group held on the evening of May 4, 2020 to discuss response to the ongoing
COVID-19 pandemic crisis.
The online NAM Contact Group Summit on the theme of ―[x]‖. was hosted by the current Chairman of
NAM, President of Republic of Azerbaijan[y]. The objective of the Summit was to promote international
solidarity in the fight against the COVID-19 pandemic and to mobilise efforts of States and international
organisations to address the pandemic.
PM Modi‘s participation underlined India‘s longstanding commitment to the principles and values of NAM
as one of its leading founding-member. In his intervention, Prime Minister emphasized the importance of
a coordinated, inclusive and equitable response by the world to this crisis, outlining the steps India had
taken domestically and internationally, while reaffirming India‘s readiness to offer assistance in solidarity
with the Movement, to the extent possible. PM also emphasized the importance of a continued effort by
the world against other viruses, in particular terrorism and fake news.
PM Modi was joined by over 30 other Heads of State and Government and other leaders, including from
member States in Asia, Africa, Latin America and the Caribbean, and Europe.
Overall, NAM Leaders assessed the impact of COVID-19, identified needs and requirements for possible
remedies and urged action-oriented follow-up measures. Following the Summit, leaders adopted a
Declaration underlining the importance of international solidarity in the fight against COVID-19. Leaders
also announced the creation of a ‗Task Force‘ to identify needs and requirements of member States
through the establishment of a common database reflecting their basic medical, social and humanitarian
needs in the fight against COVID-19.

59. What was the theme of the Summit, redacted as [x] in the baove passage?
(a) United against COVID-19
(b) Unified World against COVID-19
(c) War against Corona
(d) Transparency and Unification during Pandemic

60. Who is the current Chairman of NAM redacted as [y] in the above passage.
(a) Josip Broz Tito
(b) Gamal Abdel Nasser
(c) HouariBoumediène
(d) IlhamAliyev

61. In the abovementioned Summit the leaders also commemorated the International day of__________?
(a) Multilateralism and Diplomacy for Peace.
(b) Gender Equality
(c) Global Peace
(d) Commemoration in Memory of the Victims of the Holocaust

62. Which among the following Conferences is the predecessor of NAM?


(a) Bretton Woods Conference
(b) Bandung Conference
(c) Yalta Conference
(d) Maastricht Conference

63. How many members are there in NAM currently?


(a) 120
(b) 122
(c) 125
(d) 130

Head Office: 127, Zone II, MP Nagar, Bhopal |+91-9111555433|www.legaledge.in Page 14 of 35


MOCK CLAT #04

Passage (Q.64-Q.68): For the first time in its history, the United Nations will not have the hustle and
bustle that goes with the convergence of world leaders, ministers, diplomats, civil society members and
journalists for a General Assembly this year.
Rather, heads of states and governments have been asked to send pre-recorded video statements that
will be played out as the UN General Assembly (UNGA) session goes ―virtual", thanks to covid-19
pandemic sweeping the globe.
According to the programme schedule, UNGA will begin on 15 September. The General Assembly
meeting to commemorate the [x]th anniversary of the UN will take place on 21 September with a
declaration formally adopted at the end of it. The General Debate– which would under normal
circumstances have seen leaders and ministers from 193 UN member states address a global audience
from behind the iconic lectern in the General Assembly hall -- will start on 22 September.
Besides this, a bio-diversity summit is scheduled for 30 September, a meeting of the GA on the 25th
anniversary of the Fourth World Conference on Women is on 1 October while another to promote the
International Day for the Total Elimination of Nuclear Weapons is on 2 October. World leaders have been
requested to submit their recorded messages for these events as well.
A well documented anecdote is how former prime minister Rajiv Gandhi met former Turkish leader
TurgutOzal in an impromptu setting at the UN in 1985 that led to an exchange of visits and an unfreezing
of ties between India and Turkey fraught over New Delhi‘s support to the Greek Cypriots and Ankara‘s
support to Pakistan over Kashmir.

64. Where was the UN General Assembly supposed to take place?


(a) New York
(b) Hague
(c) Switzerland
(d) Geneva

65. Who will be the chairman of this session of UNGA?


(a) Tijjani Muhammad-Bande
(b) MiroslavLajčák
(c) VolkanBozkır
(d) María Fernanda Espinosa

66. Which UNGA session is this redacted as [x] in the above passage?- 75
(a) 74
(b) 75
(c) 78
(d) 79

67. Who is UN‘s current Secretary General?


(a) AntónioGuterres
(b) Ban Ki-moon
(c) Kofi Annan
(d) AlbaqGhali

68. Who will be India‘s next permanent representative to the UN, New York?
(a) Hardeep Singh Puri
(b) T. S. Tirumurti
(c) Indra Mani Pandey
(d) SamarendranathSen

Head Office: 127, Zone II, MP Nagar, Bhopal |+91-9111555433|www.legaledge.in Page 15 of 35


MOCK CLAT #04

SECTION – C :LEGAL REASONING

Direction (Q.69-Q.107): Read the comprehension carefully and answer the questions based on it.

Passage (Q.69-Q.73): Mistake may operate upon a contract in two ways. It may, firstly, defeat the
consent altogether that the parties are supposed to have given, that is to say, the consent is unreal.
Secondly, the mistake may mislead the parties as to the purpose which they contemplated.
Where both the parties to an agreement are under a mistake as to a matter of fact essential to the
agreement, the agreement is void. However an erroneous opinion as to the value of the things which
forms the subject –matter of the agreement, is not deemed a mistake as to a matter of fact. An
agreement upon the same thing in the same sense is known as true consent or consensus ad idem, and
it is the root of every contract. Two or more person are said to consent when they agree upon the same
thing in the same sense.
A contract is said to be void because of mistake when:
 Both the parties to an agreement are mistaken,
 Their mistake is as to a matter of fact, and
 The fact about which they are mistaken is essential to the agreement.
A contract is not void because it was caused by a mistake as to any law in force in India; but a mistake as
to a law not in force in India has the same effect as a mistake of fact.
Mistake as to identity occurs when one of the parties represents himself to be some person other than he
really is. There can be a mistake of identity only when a person bearing a particular identity exists within
the knowledge of the plaintiff, and the plaintiff intends to deal with him only. If the name assumed by the
accused is fictitious there will be no mistake of identity.

69. A, being entitled to an estate for the life of B, agrees to sell it to C. B was dead at the time of the
agreement, but both parties were ignorant of the fact. Decide the validity of the contract?
(a) The agreement is not void since B was alive when the contract was formed.
(b) The agreement is not void since the essential condition of the agreement was an estate and not the
presence of B.
(c) The agreement is valid and enforceable.
(d) The agreement is void since both the parties were ignorant of the death of B, which was an essential
fact of the contract.

70. A person called Hillary, brother of Ron, represented himself as Ron, and thereby induced a Government‘s
agent to contract with him. The government agent intended to contract only with Ron and not with Hillary
and Ron knew this. The government‘s offer was meant for Ron and Hillary posing as Ron accepted it.
Decide whether the consent of government‘s agent is valid or not?
(a) The consent is valid since the government agent could have easily figure out that it was Hillary and
not Ron.
(b) The consent given by the government‘s agent is not valid since he was deceived by the brothers
(c) This is a valid contract since the party to a contract is not an essential fact.
(d) The contract is not void since the brother belongs to same family and contracting with either of the
brothers does not affect the conditions of the contract.

71. Jethalal had taken over the business of one Brocklehurst. Heeralal used to deal with Brocklehurst and
not knowing of the change sent him an order for certain goods. The order was received by Jethalal who
sent the goods. Heeralal came to know of the change only when he received an invoice and by that time
he had already consumed the goods. Heeralal had a set-off against Brocklehurst and, therefore refused
to pay the price. Jethalal sued him:
(a) Heeralal is liable to pay since he consumed the goods and hence bound to pay.
(b) Heeralal is not liable to pay since he never contracted with Jethalal.
(c) Heeralal is not liable since his consent is not valid because he never consented to a contract with
Jethalal.
(d) Heeralal is liable to pay since the party to the contract is not an essential condition.

Head Office: 127, Zone II, MP Nagar, Bhopal |+91-9111555433|www.legaledge.in Page 16 of 35


MOCK CLAT #04

72. A man called North, entered the Jeevan‘s shop and selected some pearls and some rings worth Rs.
3000. He produced a cheque book and wrote out a cheque for the amount. In signing it he said: ‗You see
who I am, I am Sir George Bullough‘ and finding on the reference to a dictionary that Sir George Bullough
lived at the address mentioned, Jeevan let him have a ring. But before the fraud was discovered he
pledged the ring to Shyam, who advanced money bona fide, and without notice. Jeevan sued Shyam for
the ring or its value. Decide whether Shyam is liable or not?
(a) Jeevan will neither get the ring nor its price since he sold it with the greedy intention to make
excessive profit.
(b) North is liable to pay the price of the ring or return it to Jeevan since it was he who committed the
fraud.
(c) Since North himself does not have good title over the ring he cannot sell it to Shyam and hence
Shyam is liable.
(d) Shyam is not liable to return the ring or pay its equivalent price to Jeevan since was innocent on his
part while purchasing it.

73. Thomas received orders in writing from a fraudulent man, called Blenkarn. The order paper had a printed
heading: ‗Blenkarn & Co.‖. There was a well know and respectable firm, named ‗Blenkiron & Co.‖ in the
same street. Thomas believing that the orders had come from this firm, sent a large quantity of
handkerchiefs. Blenkarn received the goods and disposed them off to John, who acted in good faith and
purchased them. Decide the right of Blenkarn over the goods:
(a) Blenkarn has title over the goods even though Thomas never intended to deal with him.
(b) Blenkarn has no title over the goods since Thomas never intended to deal with him and hence there
was no contract between them.
(c) Blenkarn has title over the goods since he had paid for them.
(d) There is contract between the parties since the essential condition of contract, i.e. delivery and
payment of good is met.

Passage (Q.74-Q.79): The relevant sections of the IPC are Section 269 covering negligent act likely to
spread infection of disease dangerous to life, Section 270 covering malignant act likely to spread
infection of disease dangerous to life, Section 271 covering disobedience of quarantine.
The penalties for the above offences are simple or rigorous imprisonment extendable up to 6 months or
fine or both; simple or rigorous imprisonment extendable up to 2 years or fine or both; and simple or
rigorous imprisonment extendable up to 6 months or fine or both, respectively.
While the offences under Section 269 and 270 are cognisable and bailable, offence under Section 271 is
non-cognisable and non-bailable. This would mean that unlike in bailable offences where the accused
has a right to bail, they would be at the discretion of the court in seeking a bail. Relating these general
provisions to the core theme of the article, it provides a resounding affirmation that the law does not in
any manner authorise the police to use corporal force.
Disaster Management Act, 2005
The first relevant provision of the act is Section 51. The provision has twin aspects to it. Firstly, it is
attracted by the persons who leave their homes to pursue non-essential work. In the words of Section 51,
the conditions and punishment prescribed are –
Condition: Whoever, without reasonable cause (a) obstructs any officer or employer (b) refuses to comply
with any direction. Punishment: Imprisonment of a term upto 1 year or fine (not prescribed under the
provision) or both. Secondly, the latter part of Section 51, in present conditions, is attracted by persons
who are tested positive for the Coronavirus but run away from quarantine.
In the words of Section 51, the conditions and punishment prescribed are –
Condition: Whoever, without reasonable cause (a) obstructs any officer or employer (b) refuses to comply
with any direction causing loss of lives or imminent danger thereof. Punishment: Imprisonment of a term
upto 2 years.

Head Office: 127, Zone II, MP Nagar, Bhopal |+91-9111555433|www.legaledge.in Page 17 of 35


MOCK CLAT #04

74. Kunjbihari returned from Dubai in February, 2020 and was diagnosed with COVID-19 after he showed
symptoms of the disease. He was admitted to the hospital and recovered within 15 days after which he
was sent back home and was required to self-quarantine for the next 14 days. Kunjbihari however, was
craving street food and ventured out without knowing that the disease had recurred. He infected around
250 people with the disease. Decide
(a) Kunjbihari is liable under Section 269 and 271 of the Indian Penal Code
(b) Kunjbihari is liable under Section 270 and 271 of the Indian Penal Code
(c) Kunjbihari is not liable for any offence as he had recovered from the disease
(d) None of the above

75. Please refer to the facts mentioned above. Kunjbihari upon his return from Dubai bribed the police
personnel and evaded the mandatory screening for COVID-19 for he suspected a positive report. He
roamed around the entire city of Delhi spreading the infection to about 200 people.
(a) Kunjbihari is liable under Section 269 of the Indian Penal Code
(b) Kunjbihari is liable under Section 270 of the Indian Penal Code
(c) Kunjbihari is not liable for any offence as he would recover from the disease
(d) None of the above

76. Apurv returned from the United States. On his way out of the airport, he tried to escape screening but
was caught. When he was screened, he was found positive. However, he broke his quarantine and ran
away. He was booked by the police for offending provisions of the Indian Penal code. In addition, he was
also booked for breaking self-quarantine. Apurv‘s lawyers approached the court seeking bail and
asserted that it was their right to get bail.
(a) Apurv can seek bail as a right for it was his right to get bail under Section 269/270 of IPC
(b) Apurv cannot seek bail as a right for he is a heinous public offender who has caused significant
problems
(c) Apurv can seek bail as a right for he needs medical care at this point in time
(d) Apurv cannot seek bail as a right for he doesn‘t have a right under Section 271 of IPC

77. In the wake of the COVID19 outbreak, there has been a lockdown imposed all over the country. There
have been instances of police personnel using innovative methods to enforce such lockdowns. While
some police personnel were spotted getting citizens to do push up, some others have been spotted
getting citizens to clean roads. Decide
(a) The police personnel are justified to impose such punishments for they do not involve the use of
corporal force.
(b) The police personnel are not justified to impose such punishments for they involve the use of corporal
force
(c) The police personnel are justified to impose such punishments for they are required in the interest of
the nation
(d) None of the above

78. Please refer to the fact scenario in 6.4. The police personnel were spotted hitting a person with a lathi
after an argument happened when the person refused to head back home during the lock down.
(a) The police personnel are justified to impose such punishments for they do not involve the use of
corporal force.
(b) The police personnel are not justified to impose such punishments for they involve the use of corporal
force
(c) The police personnel are justified to impose such punishments for they are required in the interest of
the nation
(d) None of the above

Head Office: 127, Zone II, MP Nagar, Bhopal |+91-9111555433|www.legaledge.in Page 18 of 35


MOCK CLAT #04

79. Ajinkya wanted to go out of his home during the COVID-19 lock down to see the fresh Bangalore Cherry
Blossoms everyone was talking about. It happens to be that this was a once in a 12 year opportunity and
missing this chance would mean waiting for another 12 years. He put on his suit, mask and gloves and
set out to leave. The police personnel outside Cubbon Park stopped Ajinkya. Ajinkya refused to comply.
Decide
(a) Ajinkya violated Section 51 of the Disaster Management Act, 2005 for this cause is unreasonable
(b) Ajinkya did not violate Section 51 of the Disaster Management Act, 2005 for the cause is reasonable
(c) Ajinkya needs to understand the gravity of the situation
(d) Ajinkya had taken all necessary precautions. Cause cannot be stated to be unreasonable in that case

Passage (Q.80-Q.85): The term ‗Force Majeure‘ appeared in the common law world in the 1900s and
was borrowed from the Napoleonic Code (see Lebeaupin v Crispin [1920] 2KB 714), although its origins
can be traced back to Roman law.
Under Indian law, one of the first decisions to deal with the concept of force majeure was the Madras
High Court decision in Edmund Bendit And Anr. vs Edgar Raphael Prudhomme. In this case, the Court
cited with approval the passage from Matsoukis v. Priestman and Co, wherein the definition given by an
eminent Belgian lawyer of force majeure as meaning "causes you cannot prevent and for which you are
not responsible", was adopted. The test for seeking to rely on a force majeure clause is that the event
and the non-performance were due to circumstances beyond a party's control. Thus, force majeure will
not include economic problems like insufficient funds. Another condition that needs to be fulfilled is that
there were no reasonable steps that could have been taken to avoid or mitigate the event or its
consequences. The party seeking to rely on the clause may also need to show it was not aware, at the
time of entering the contract, that the circumstances giving rise to the event of force majeure was likely to
occur.
If the clause refers only to performance of obligations being prevented by the relevant event, then a
party may not be able to rely on the clause if its performance has been made more difficult or delayed,
but not completely prevented (in other words, it can still perform, but it is more difficult to do so and/or it
cannot perform as expected). The fact that it is more expensive to perform will also not normally entitle
reliance on the force majeure clause (so a rise in underlying costs or expense will not normally be treated
as a force majeure event). The fact that it is more inconvenient and/or more difficult to perform will also
not normally entitle reliance on the clause. If the difficulty was due to circumstances outside a party's
control and was so great that no reasonable person in similar circumstances would be likely to overcome
it, then that might be an exception.

80. JBM Environment Ltd. has been asked to construct a solid waste management plant in New Delhi for
power generation in the year 2019. As they seek to move forward with the procurement of raw material
for construction of the plant, the NGT bans all construction activity in the area due to increasing pollution.
JBM stops all work. Decide
(a) JBM can seek the benefit of a force majeure clause for construction was stopped for circumstances
beyond their control i.e. pollution in Delih NCR
(b) JBM cannot seek the benefit of a force majeure clause for they could still have procured all raw
material
(c) JBM can seek the benefit of a force majeure clause for construction was stopped for circumstances
beyond their control i.e. NGT order.
(d) None of the above

81. Please refer to facts abovementioned. The construction ban was lifted after 2 months and JBM sought
an extension citing force majeure. They sought for 4 months extension i.e. 2 months for procurement of
raw material and 2 months for construction. However, the same was refused to them by the contracting
party. Decide
(a) JBM is entitled to 4 months of extension for force majeure reasons
(b) JBM is entitled to two months of extension for they could have taken steps to mitigate the delay.
(c) JBM is entitled to no extension for they could have still carried out with the construction.
(d) JBM should drag the party to court.

Head Office: 127, Zone II, MP Nagar, Bhopal |+91-9111555433|www.legaledge.in Page 19 of 35


MOCK CLAT #04

82. SuperNova is an engineering company and were constructing a dam on Teesta river when a ban was
placed on the dam construction due to tribal protest. During the construction ban, there was a mass
exodus of labourers for they had no livelihood left in the area. After the construction ban was lifted, it took
around 1 month for the labourers to return to the area. Since there were only a limited number of
labourers in the city, engaging their service became quite expensive. SuperNova preferred to wait it out
for the labourers to return. When they couldn‘t meet the deadline, they cited force majeure and sought an
extension of 1 month for the time lost.
(a) SupreNOVA should be given the extension as the delay was caused due to circumstances beyond
their control
(b) SUPERNOVA should not be given extension as the construction ban had now been lifted
(c) SUPERNOVA should be given the extension as cost overruns were a major concern
(d) SUPERNOVA should not be given extension for mere difficulty to perform is not force majeure

83. Please refer to the facts above. During the construction ban, there was mass exodus of labourers. The
mass exodus of labourers meant that there were no labourers left in area and thus no construction work
could be carried out whatsoever for the next 1 month till the labourers came back. SUPERNOVA upon
not being able to fulfill the deadline asked for extension of 1 month citing force majeure.
(a) SUPERNOVA should be given the extension as the delay was caused due to circumstances beyond
their control
(b) SUPERNOVA should not be given extension as the construction ban had now been lifted
(c) SUPERNOVA should be given the extension as cost overruns were a major concern
(d) SUPERNOVA should not be given extension for mere difficulty to perform is not force majeure

84. BB is an aircraft manufacturing company. They often depend on cheaper services of mechanical
engineers from third world countries for their job. However, in 2020 due to the suspension of H1B1 Visas,
a significant shortage was seen in the influx of such engineers. The cost due to the shortage had driven
upto 10 times. The manufacturing work being engineer heavy was severely hit due to such high prices
and the Aircrafts Manufacturing Association of New Jersey (of which BB was a member) collectively
decided to ban manufacturing for 2 months feraing cost significant overruns. BB complied too. When they
couldn‘t meet the deadline, they cited force majeure and sought an extension of 2 months for the time
lost.
(a) BB should not be given extension for mere difficulty to perform is not force majeure
(b) BB should be given an extension for everyone else situated in the same position as them stopped
construction.
(c) BB should not be given extension as the construction ban had now been lifted
(d) BB should be given the extension as cost overruns were a major concern

85. Please refer to the facts above. Cost of engineers due to the shortage had driven upto 10 times. Multiple
sectors were severely hit due to such high prices. The Local Dam Builders Association of New Jersey
passed a resolution collectively decided to ban all manufacturing for 2 more months fearing cost
significant overruns. BB found this as an opportunity to halt construction and save money. When they
couldn‘t meet the deadline, they cited force majeure and sought an extension of 2 months for the time
lost.
(a) BB should not be given extension for mere difficulty to perform is not force majeure
(b) BB should be given an extension for everyone else situated in the same position as them stopped
construction.
(c) BB should not be given extension as the construction ban had now been lifted
(d) BB should not be given extension as they are not situated similarly as the Local Dam Builders
Association of New Jersey.

Head Office: 127, Zone II, MP Nagar, Bhopal |+91-9111555433|www.legaledge.in Page 20 of 35


MOCK CLAT #04

Passage (Q.86-Q.90): Right to Privacy has always been a contentious issue. The COVID-19
pandemic has brought new considerations to the table. In the coordinated efforts of authorities across
the globe to tackle this pandemic we see a sacrifice of an individual‘s right to priva cy.
Instances of a heightened tracking of an individual‘s location, mass surveillance, applications to keep
track of your personal movement, use of travel history have raised pertinent privacy implications.
Various states have tapped phone records, CCTV footages, phone GPS and published detailed
timelines and quarantine lists of the patients and officials.
A patient‘s right to privacy with regard to their personal details was recognized in the case of Mr. X v.
Hospital Z. The court opined that the patient had the right to confidentiality and privacy as regards their
personal details; however, this right was not absolute. Further, the Apex Court has both in Puttaswamy I
and Puttaswamy II crystallized the importance of informational privacy i.e. data protection.

The Court held that any infringement of a right to privacy by the government must be reasonable a nd
proportionate, and must satisfy the following (i) the restriction must be effected through a law which
pursues a legitimate state aim, (ii) has a reasonable nexus between the objects and means to achieve
them, and (iii) is the least intrusive means to achieve the state aim.
Further, the Personal Data Protection Bill provides that the consent of the person whose personal
data is collected (Data Principal) is necessarily to be obtained by any person or the government (Data
Fiduciary). The personal data collected must be used only for a specific, clear and lawful purpose for
which the consent has been obtained. The personal data must also be used in a fair and reasonable
manner. The PDP Bill also provides that under emergent circumstances the Data Fiducia ry is exempt
from taking the consent of the Data Principal, provided that the collection and processing of personal
data is authorized under law. Though just a Bill, this can be a guiding factor in deciding the use of a
person‘s information. The state at no point in time can act disproportionately even in a pandemic.

86. Mr. Bakliwala is a doctor of immense repute. He is known for his experience and service in the field of
sexually transmitted diseases such as HIV. Karan approaches Mr. Bakliwala and gets tested for HIV and
the result turns out to be positive. Mr. Bakliwala informs Karan that with the advancements in medical
science the situation can be sustained if not completely cured. A few days later a group of researchers
come to work under Dr. Bakliwala and he shares the instances of his patients with the team, which also
includes Karan. In the end result published, Karan‘s name is included in the report. Decide:
(a) Karan‘s right to privacy has been violated by Mr. Bakliwala.
(b) Karan‘s right to privacy has not been violated by Mr. Bakliwala as the same was for research
purposes and shall in turn help the majority of public at large and thus was justified.
(c) Karan‘s right to privacy in such a situation can be curtailed as HIV is a disease which can have a
bearing on the society at large and thus it was important to release the same.
(d) Karan‘s right to privacy has been violated as the information of such a nature is a part of informational
privacy.

87. Findcaller is a mobile based application which helps individuals to intercept who is calling them. It
requires individuals to register their details and personal information and processes the same to intercept
calls. On 4th May, 2020 Findcaller accidentally revealed the details of 4 lakh customers on various
websites. A probe into this event later revealed that Findcaller was selling such data to companies who
created their databases from such information. Assuming that the PDP Bill is now an Act, decide whether
there was a breach of privacy.
(a) No, there is no breach of privacy as this right is only enforceable against the state.
(b) No, as the selling of data was for commercial purposes and the same is justified on business
grounds.
(c) Yes, as there was a public release of such information. However, only those consumers whose
information has been revealed on websites can contest a breach.
(d) Yes, there has been a breach of privacy on part of Findcaller.

Head Office: 127, Zone II, MP Nagar, Bhopal |+91-9111555433|www.legaledge.in Page 21 of 35


MOCK CLAT #04

88. The Karnataka State Government has been ramping up its activities to tackle the COVID-19 pandemic.
Several steps have been taken to this effect and one of these steps include preparing a list of people who
have been asked to self-quarantine and uploading the same on the website of the government. This had
been duly informed to the individuals as a precondition of returning to the state from abroad or
elsewhere. As soon as the quarantine period for an individual will end his or her name would be removed
from the list. But as a result of this the individuals whose name was included in the list were ostracized.
Will such a situation amount to violation of right to privacy?
(a) No, such a situation shall not be a violation of the right to privacy as it is in the larger public interest.
(b) No, such a situation will not be a violation of the right to privacy as the restriction has been effected
through a legitimate law and is in consonance with the goal it seeks to achieve.
(c) Yes, it is a breach of privacy as it was not effected through any legislation passed by the government,
rather it was an arbitrary act of the authorities.
(d) Yes, it is a breach of the right as state cannot restrict the right in such an arbitrary manner.

89. Ram is a student who duly fills out the CLET (Common Law Entrance Test Form). He fills in information
of sensitive nature such as his contact details, address, caste etc. into the form and successfully registers
for the same. This is the only form he has filled for colleges. After a few days Ram starts getting
messages from other private universities about their law program and other coaching institutes about
their different courses. He is quite baffled as he never shared this information with anyone on any
platform. He approaches the High Court contending the violation of Right to Privacy and contends that in
all probability his data has been compromised by CLET. Decide:
(a) His contention is correct as all factors reasonably indicate that the information has been accessed via
the CLET database.
(b) His contention is correct as his personal data was for a specific purpose and such use of this data
by other universities and coaching‘s is not in consonance with law.
(c) His contention is wrong as CLET can give access to other universities in the legal domain to take
information and develop their database. This however should have been duly communicated to Ram.
(d) His contention is correct because the principles of proportionality have been violated by the CLET.

90. Mr. Bakliwala also finds out that Karan is getting married in a few days. He also knows that Karan has not
informed anyone about his HIV status and neither does he intend to inform anyone. Mr. Bakliwala goes
on to find Karan‘s fiancée and reveals this information to her. Karan says that this is a violation of his right
to privacy. Mr. Bakilwala argued that this was in furtherance of the fiancée‘s right to healthy life under
Artcile 21. Decide:
(a) This is not a violation of right to privacy as this would have had severe impacts on the life of the wife
hence that right i.e. right to health shall outweigh Karan‘s right to privacy.
(b) This is a violation of right to privacy as personal autonomy must be given to Karan.
(c) This is a violation of right to privacy as this information can have immense impact on Karan‘s life and
thus Karan should have the right over such information.
(d) This is not a violation as right to a healthy life will outweigh the right to privacy of Karan.

Passage (Q.91-Q.96): Indian hospitals have been held liable for their services- rather lack of services-
individually or vicariously. They can be sued for negligence either in Criminal Courts, Civil Courts or
Consumer Forums. The Supreme Court has held that every doctor ―has a duty to act with a reasonable
degree of care and skill‖ in the case of State of Haryana v. Smt. Santra.
The liability of a medical practitioner does not merely arise when the patient has suffered an injury; it
arises when the injury is a direct consequence of the conduct of the doctor when he failed to exercise
reasonable care. In other words, a doctor is not liable for every injury suffered by the patient. First,
existence of a duty of care by the doctor towards the patient has to be established and then the patient
must prove breach of such a duty. In case there was no breach or the injury was not a direct
consequence of the breach, the doctor will not be liable. It was held in Calcutta Medical Research
Institute v. Bimalesh Chatterjee that the onus of proving negligence and the resultant deficiency in
service was clearly on the complainant.
The liability of a hospital in cases of medical negligence could be direct or vicarious. Direct liability in this
sense would mean a deficiency in the services provided by the hospital thus making it unsafe and not

Head Office: 127, Zone II, MP Nagar, Bhopal |+91-9111555433|www.legaledge.in Page 22 of 35


MOCK CLAT #04

suitable for treatment. Vicarious liability, on the other hand, would refer to the liability of the hospital as an
employer for the negligent acts of its employees.
The employer is responsible not only for his acts and omissions but also for those of his employees, as
long as such acts occur within the course and scope of employment. This liability is based upon the
maxims “respondent superior” which means ―let the master answer‖ and “qui facit per alium facit per
se” which means ―He who acts through another does the act himself.‖
An exception to the above principle is seen in the ―borrowed servant doctrine‖ according to which the
employer shall not be liable for acts of an employee when that employee is working under the direct
supervision of another employer.

91. Professor and Nairobi conceived a baby through IVF. In her 26 th week of pregnancy, Nairobi developed
gestational diabetes. She was prescribed with metformin. After 39 weeks, Nairobi went into tough labour
and gave a birth to baby boy. The baby was born with permanent physical disability and had to be
resuscitated. It was later found that the disability is a side effect of metformin (a drug that should not be
consumed by pregnant women). Professor and Nairobi decided to sue the doctor. Decide.
(a) The doctor will be held liable as this is a clear case of negligence. It was the duty of the doctor to
inform about the side effects of the drug.
(b) The breach of duty of care by doctor while prescribing a wrong medicine makes him accountable for
medical negligence.
(c) The hospital, along with doctor will be liable for medical negligence.
(d) The couple will not be able to proceed as it was their duty to consult an endocrinologist for diabetes
as it cannot be expected for a gynaecologist to prescribe correct medicine for diabetes.

92. Mr. Batra after getting his heart surgery done from KEM Hospital, alleged that he was treated badly by
the staff of the hospital during his stay at the hospital. He also alleged that Dr. Mahesh did not possess
the required skills for conducting a surgery and when the first procedure failed, to rectify his mistake he
performed a corrective surgery. Hospital and Dr. Mahesh denied all the allegations against them. As per
the given passage, with whom does the onus of proof lies?
(a) The onus of proof lies with defendant. Therefore, in the present case the doctor and the hospital
management should prove their innocence.
(b) The onus of proof lies with the accused. Therefore, in the present case Mr Batra should prove
negligence.
(c) The onus of proof lies with the plaintiff. Therefore it‘s the duty of Mr. Batra to provide evidence.
(d) The onus of proof lies with the deponent. Therefore it will be the duty of the Hospital and Dr. Mahesh
to provide evidence.

93. When Mrs. Menon went for her spinal cord surgery, Dr. Knight failed to convey her the risk involved in the
procedure nor did he discuss the outcomes of failure of the surgery. After the surgery, Mrs Menon was
left disabled with her lower part of the body. Mrs. Menon wishes to take your advise. Kindly advise her.
(a) Mrs. Menon will succeed against Dr. Knight as he lacked possession of required set of skills for a
surgeon.
(b) Mrs. Menon will not succeed as she never asked the doctor about the possible ill outcomes of the
surgery.
(c) Mrs. Menon will succeed as Dr. Knight owed a duty of care towards his patients to keep them well
informed and he breached his duty of care resulting in negligence.
(d) Mrs. Menon will not succeed as every surgery has a possibility of failed outcome and this should not
be regarded as a breach of duty at the hands of the doctor.

Head Office: 127, Zone II, MP Nagar, Bhopal |+91-9111555433|www.legaledge.in Page 23 of 35


MOCK CLAT #04

94. Dr Umang and Dr Samara are the resident doctors at DAIIMS, Bangalore. Both of them are appointed
with the neo natal care unit in the hospital and are assigned the duty to monitor the babies in the unit who
are under critical care. Once on duty, Umang and Samara indulged in playing ludo and forgot to switch
on the oxygen supply, resulting in death of four children. A case was brought against the hospital decide.
(a) Hospital will not be held liable as the case doesn‘t qualify under vicarious liability.
(b) The hospital will be vicariously held liable as the medical negligence took place during the course of
employment of two doctors.
(c) Dr Umang and Dr Samara will be held liable and not the hospital.
(d) The hospital will be held liable and compensation should be paid by both the hospital as well as the
two doctors.

95. Which among the following cases are not justiciable?


i. Mistake done by anesthesiologists resulting in permanent brain damage or death.
ii. A botched procedure involving a large ethanol injection causing a child‘s face and nose to deteriorate,
requiring skin from his forehead and ear to repair it.
iii. Discharging Covid 19 suspect without conducting proper tests at the hospital quarantine centre.
iv. Person injuring himself after falling in the hospital premise due to wet floor.
v. Patient forgetting that he has already taken his medicine, consumes another dose resulting in high
blood pressure due to over dose of medicine.
vi. An obstetrician in Mumbai fails to perform a C-section in a timely manner, resulting in serious injuries
to the baby.
(a) Only I, II and III
(b) Except V, VI, and IV
(c) Except IV and V
(d) Only I, II, IV and V

96. Somu is suffering from kidney failure and requires dialysis every week. When he went to get his dialysis
done at AIIMS hospital, he caught corona virus infection and was later tested positive for Covid 19. He
decided to sue AIIIMS hospital for infecting him with Covid 19. Decide.
(a) Somu will succeed as AIIMS hospital breached its duty of care by failing to curtail the infection.
(b) Somu will succeed as AIIMS hospital failed to have separate quarantine centre for Covid 19 patience.
(c) Somu will not succeed as hospitals are breeding grounds of infections and one cannot do much about
it.
(d) Somu will not succeed as him catching corona infection cannot be regarded as breach of duty on the
part of the hospital.

Passage (Q.97-Q.102): To use insanity as a legal excuse, the defendant has to show that he/she lacked
the capacity to understand that the act was wrong, or the capacity to understand the nature of the act.
The logic of the insanity as a defence goes back to the idea of mensrea and culpability. The foundation of
this law was first laid in the M'Naughton case by the House of Lords in 1843. The basis of the
M'Naughtoncase is the inability to distinguish right from wrong. The basic idea is that some people, under
the duress of a mental disorder, cannot control their actions despite understanding that the action is
wrong.
Section 84: Act of a person of unsound mind.—Nothing is an offence which is done by a person who, at
the time of doing it, by reason of unsoundness of mind, is incapable of knowing the nature of the act, or
that he is doing what is either wrong or contrary to law.
So it falls upon the accused to prove his insanity at the time of offence. It needs to be proved by the
accused that because of the accused's unsoundness of mind, he was incapable of knowing the nature of
the act or that the act was contrary to provisions of law, or was wrong.
Infancy:
Infancy is a legal incapacity to be held responsible for a crime due to the age of the perpetrator. There is
a legal incapacity for the crime under seven years of age. DoliIncapaxis a presumption of law which
provides that child has no discretion to distinguish right from wrong, thus criminal intention does not arise.
Section 82: Act of a child under seven years of age.—Nothing is an offence which is done by a child
under seven years of age.

Head Office: 127, Zone II, MP Nagar, Bhopal |+91-9111555433|www.legaledge.in Page 24 of 35


MOCK CLAT #04

Section 83: Act of a child above seven and under twelve of immature understanding.—Nothing is an
offence which is done by a child above seven years of age and under twelve, who has not attained
sufficient maturity of understanding to judge of the nature and consequences of his conduct on that
occasion.
The essential ingredients of Sections 82 and 83 are that children under seven years of age is
doliincapax, i.e. he is incapable of committing a crime and cannot be guilty of any offence.

97. On her 7th birthday, Riddhima had costume party in which dress like a princess while the others came for
her class dressed as different Disney characters. One of the kids who was dressed like a pirate got into
an argument with a kid dress like the prince. In a pretend sword fight the pirate impaled the shoulder of
the prince with his costume‘s sword which had a pointy end. Would the child be convicted?
(a) Yes, as they were above the age of 7 years.
(b) No, as they were under the age of 12 and did not understand the nature of their activity.
(c) Can‘t say, as Riddhima was 7 but the kids could have been older or younger at the party.
(d) No, as they were merely playing and did not intend to cause any harm to each other, they also
wouldn‘t have known the risk of a costume.

98. A person Sarthak was prone to epileptic fits. During one of these situations he managed to knock over a
vase on the head of a passerby from his balcony. Being grievously injured, the spouse of this passerby
filed suit for bodily harm injury again Sarthak. Decide.
(a) The suit will not sustain as the spouse cannot file a complaint on behalf of the injured.
(b) The suit will not sustain as it was an accident.
(c) The suit will not sustain as he was not sane in that moment and can plead insanity.
(d) The suit will not sustain as the passerby should have been more cautious before walking underneath
someone‘s balcony.

99. A Six-year-old boy, Rohan after watching a series of criminal drama on television became extremely
intrigued with the concept of crime. One day when he was really upset after an argument with his mother
he decided to kill her in her sleep completely aware of the consequence. Can Rohan be convicted for
Murder?
(a) Yes, as he had complete understanding and maturity of the consequences.
(b) No, as he was under less than 7 years of age.
(c) Yes, as he acted out of provocation and anger and plotted it with intention.
(d) No, as a minor cannot be blamed for any offence, it falls under doli incapax.

100. After finding out that his wife was having an affair with their neighbor Mr. Sharma went into depression.
One day after reconsidering the situation in a fit of rage, decided to physically harm either one of them. In
his attempt for vengeance, he ended up causing the death of his neighbor. He later claimed before the
court that he had lost his sanity and that hour and hence should not be found guilty. Decide.
(a) Yes, as he was not thinking straight and had lost his sanity to make prudent decisions.
(b) No, as anger is not a ground of claiming insanity. It has to be a medically proven one.
(c) No, as he had already plotted revenge before got a fit of rage.
(d) Yes, as he have plotted something in his sanity, but he only acted on it in moments of insanity.

101. A child of 14 years of age found a lost ring in a park and decided to keep it as he found it really pretty.
Thinking that it belongs to nobody else he decided to sell the same and keep the money for himself. Has
a child committed an offense?
(a) Yes, as he knew the consequence of his action.
(b) Yes, as he was above the age of 14 years and hence can be held liable for any an offence regardless
of his maturity.
(c) Yes, as he may not have had any motive when he picked the ring, but he later sold it for wrongful
gain.
(d) Yes, as he should have handed it to any adult or the police as he is old enough to know is duties and
responsibilities.

Head Office: 127, Zone II, MP Nagar, Bhopal |+91-9111555433|www.legaledge.in Page 25 of 35


MOCK CLAT #04

102. A school took their students on a field trip to a museum. Among the student of class 9th was Tina, who
was a kleptomaniac. Unable to contain herself she managed to swipe what are the pieces of jewelry from
under the display. She was called later by the authorities and charged for theft. Should she be convicted?
(a) Yes, as was older than the age of 12 years clearly.
(b) No, as he was a kleptomaniac and can use the defense of insanity to get an acquittal.
(c) Yes, as even though she was older than 7 years of age, she did not have the maturity or
understanding of an adult.
(d) Can‘t say as the facts do not mention her age or her mental status.

Passage (Q.103-Q.107): Religion is a matter of belief or faith. The constitution of India recognizes the
fact, how important religion is in the life of people of India and hence, provides for the right to freedom of
religion under Articles 25 to Article 28. The Constitution of India envisages a secular model and provides
that every person has the right and freedom to choose and practice his or her religion. In a number of
cases, the Apex Court has held that secularism is the basic structure of the Constitution, the most
important being the Kesavananda Bharati case.
Secularism means developing, understanding and respect for different religions. It is believed that the
word ‗Secularism‘ has its origin in late medieval Europe. In 1948, during the constituent assembly debate,
a demand was made by the KT Shah to include the word ‗Secular‘ in the Preamble to the Constitution.
The members of the assembly though agreed to the secular nature of the constitution but it was not
incorporated in the Preamble. Later, in 1976 the Indira Gandhi government enacted the 42nd
Amendment Act and the word ‗Secular‘ was added to the Preamble. The 42nd Amendment Act also
known as the ‗Mini Constitution‘, is the most comprehensive amendment to the Constitution.
Article 25 of the Constitution guarantees freedom of religion to all persons in India. It provides that all
persons in India, subject to public order, morality, health, and other provisions:
 Are equally entitled to freedom of conscience, and
 Have the right to freely profess, practice and propagate religion.
It further provides that this article shall not affect any existing law and shall not prevent the state from
making any law relating to:
 Regulation or restriction of any economic, financial, political, or any secular activity associated with
religious practice.
 Providing social welfare and reform.
 Opening of Hindu religious institutions of public character for all the classes and sections of the
Hindus.

103. India has been land of various customs and usages. One of them includes the marriage of women with a
tree on account of them being Mangliks. Mangliks is an astrological concept wherein any woman having
this in their chart would lead to an early death of their husband. Therefore, the curse is broken down by
undertaking the aforementioned ceremony. This tree marriage ceremony is still practiced by few people
in northern India. The Rajasthan government decides to pass a law banning this ceremony. Decide.
(a) The law poses a reasonable restriction as such superstitions should not sustain.
(b) The law is reasonable as it affects a very smaller section of the society.
(c) The law is unreasonable as it does not instigate anything to fall under the purview of being against
public interests.
(d) The law is not reasonable as it impacts the right of women to marry.

104. St. Joseph International School is administered by a Christian family. To thank the Almighty, the school
used to have certain lines from Bible recited everyday at the school assembly. Three students refused to
be a part of the process and therefore expelled. The students filed a petition that this violates their right to
religion as they belong to a religion sect which does not allow them to take part in thanking god through
different means. Decide.
(e) The petition will succeed as their right of religion was getting affected.
(f) The petition will not succeed as it was not a matter of religion but just a way of expressing gratitude.
(g) The petition will not succeed as the religion of the students puts forth unreasonable circumstances.
(h) Both (b) and (c)

Head Office: 127, Zone II, MP Nagar, Bhopal |+91-9111555433|www.legaledge.in Page 26 of 35


MOCK CLAT #04

105. The Government of Himachal Pradesh passes a law which makes the religious conversions or attempt of
conversions on account of force or allurement as a penal offence. Some Hindu leaders challenge the law
claiming it to be in violation if Article 25 on the ground that the term propagate in the aforementioned
article gives them the right to convert people into their belief system and faith. Decide
(i) The law is constitutional as the right to convert cannot be part of right to religion.
(j) The law is not constitutional as it violates the right of the leaders to propagate the religion.
(k) The law is constitutional as it talks about conversion on account of fraud and allurement.
(l) The law is constitutional as reasonable restrictions can be imposed based on the requirements of the
situation.

106. A religious sect has a practice of offering prayers five times in a day. While offering prayers their temple
used loudspeakers which was creating disturbance for the residents of the nearby area. A lot of residents
started complaining especially for the early and late hours. One such resident named Revathi filed a
complaint against the priest to the concerned administrative authorities. The priest justified by taking the
argument of freedom of religious practice. Meanwhile, the state passes a law restricting the use of
loudspeakers during prayer offerings in early and late hours. The priest moves to the court challenging
the law to be unconstitutional. Decide.
(m) The priest will succeed as it is violation of the right granted under Article 25.
(n) The priest will fail as it is a reasonable restriction on the method of the practice.
(o) The priest will fail as the state has to take care of unreasonable religious practices.
(p) The priest will succeed right of religion cannot be restricted through such means.

107. Momina is a Muslim girl who wears a headscarf as a religious symbol. She studies in a government
school and keeps it on even during class hours. The school prescribes for a school uniform and the scarf
is not a part of the said uniform. She is being asked to remove the scarf during class hours but she
denied. On such refusal she is punished by the authorities for not complying with the uniform. Decide.
(q) The fundamental right of religion has not been violated as it is just a matter of school uniform.
(r) The fundamental right of religion has not been violated as same uniform is mark of equality.
(s) The fundamental right has been violated as the scarf was a religious symbol.
(t) Both (a) and (b)

Head Office: 127, Zone II, MP Nagar, Bhopal |+91-9111555433|www.legaledge.in Page 27 of 35


MOCK CLAT #04

SECTION - D : LOGICAL REASONING

Passage (Q.108–Q.113): Before the recent encroachments of tourism and commercialization, the culture
of the Gullah communities on the Sea Islands off the southeast coast of the United States retained a
unique identity derived partially from the Islands‘ history as an area reserved for freed slaves after the
Civil War. As an almost exclusively black community, the Gullah preserved African traditions concerning
family structure and religious practices. At the same time, as a community of ex-slaves, the residents
retained several facets of the Southern life they had left behind. This mixture provided a heritage that
until recently was strong enough to sustain a vital culture. As Patricia Jones-Jackson has pointed out, the
basic unit of social life on the Sea Islands, as in West Africa, is the extended family. Since many islands
are sectioned off into family communities, kinship ties are important to one‘s acceptance into the social
structure. Membership in the extended family also affects property rights. In the traditional Gullah system,
family members do not normally buy land from one another, but acquire it by an unwritten contract known
as ―heir‘s land.‖ Rules pertaining to marriage seem to be at least as broad in scope. Common-law
marriages are considered as legitimate as marriages recorded by contract under law. Indeed, the
infrequent occurrence of divorce and separation within the Sea Island communities demonstrates the
strong cohesion of Gullah marital and familial institutions. Unlike the laws and customs relating to family
structure, the religious practices of the Sea Islanders, on the surface at least, bespeak a U.S. heritage.
Depending on the village, a Baptist or Methodist church acts as an essential social institution. Yet, in
contrast to the dualistic body-soul approach to the individual found in Christian teaching, the Gullah
believe that a person has an earthly body, a soul that returns upon death to the Divine Kingdom, and a
separate spiritual entity that can remain on Earth and influence the lives of those still living. This belief in
a ―body spirit‖ is prevalent among West African peoples, according to Jones-Jackson. She also notes the
African influences on the interaction between the minister and the congregation: The prayers and
sermons ―embody a classical, Ciceronian rhetorical style and employ sophistic ornaments capable of
divinely inspiring and passionately persuading a congregation to respond with raucous and joyous
replies.‖

108. Which of the following best expresses the main idea of the passage?
(a) Religious Beliefs of the Sea Islanders
(b) ―Heir‘s Land‖—Key to the Gullah Culture
(c) African and U.S. Influences on Gullah Life
(d) Family Structure and Property Rights and the Gullah

109. According to the passage, all of the following aspects of the culture of the Sea Islanders show the
influence of African traditions EXCEPT the
(a) family structure
(b) conception of the afterlife
(c) method of acquiring land
(d) importance of the churches as social institutions

110. It can be inferred that the institution of ―heir‘s land‖ allows the transfer of property under the terms of
(a) a formal grant by the government
(b) a marriage settlement between families
(c) an oral agreement among family members
(d) a written deed of ownership

111. On the basis of information supplied by the passage, which of the following would most likely resemble a
social experience of a Sea Islander?
(a) Dividing property in a divorce settlement under court supervision
(b) Being required to sell one‘s home because it lies in the path of a new highway
(c) Growing up in a communal household composed of unrelated adults
(d) Being given employment by a relative on the basis of one‘s standing in the family

Head Office: 127, Zone II, MP Nagar, Bhopal |+91-9111555433|www.legaledge.in Page 28 of 35


MOCK CLAT #04

112. It can be inferred that towards the Gullah, the author is:
(a) sympathetic and receptive (b) ambivalent and uncertain
(c) detached and quizzical (d) morally outraged

113. According to the passage, Patricia Jones-Jackson believes which of the following reveals an African
influence on Gullah religious services?
(a) The role of music in Gullah services
(b) The number of Gullah sermons on African topics
(c) The infrequency of divorce among Gullah families
(d) The type of vocal participation by the Gullah congregation

Passage (Q.114– Q.119): Today's developing economies use much less energy per capita than
developed countries such as the United States did at similar incomes, showing the potential for lower-
carbon growth. Adaptation and mitigation need to be integrated into a climate-smart development
strategy that increases resilience, reduces the threat of further global warming and improves
development outcomes. Adaptation and mitigation measures can advance development and prosperity
can raise incomes and foster better institutions. A healthier population living in better built houses and
with access to bank loans and social security is better equipped to deal with a changing climate and its
consequences. Advancing robust, resilient development policies that promote adaptation is needed today
because changes in the climate, already begun, will increase even in the short-term.
The spread of economic prosperity has always been intertwined with adaptation to changing ecological
conditions. But as growth has altered the environment and as environmental changes has accelerated,
sustaining growth and adaptability demands greater capacity to understand our environment, generate
new adaptive technologies and practices and diffuse them widely. As economic historians have
explained, much of humankind's creative potential has been directed at adapting to the changing world.
But adaptation cannot cope with all the impacts related to climate change, especially as larger changes
unfold in the long-term. Countries cannot grow out of harm's way fast enough to match the changing
climate. And some growth strategies, whether driven by the government or the market, can also add to
vulnerability, particularly, if they overexploit natural resources. Under the Soviet development plan,
irrigated cotton cultivation expanded in water-stressed Central Asia and led to the near disappearance of
the Aral Sea, threatening the livelihoods of fisherman, herders and farmers. And clearing mangroves —
the natural coastal buffers against storm surges to make way for intensive farming or housing
development, increases the physical vulnerability of coastal settlements, whether in Guinea or in
Louisiana.

114. Which of the following conditions of growth can add to vulnerability?


1. When the growth occurs due to excessive exploitation of mineral resources and forests.
2. When the growth brings about a change in humankind's creative potential.
3. When the growth is envisaged only for providing houses and social security to the people.
4. When the growth occurs due to emphasis on farming only.
Select the correct answer using the codes given below
(a) Only 1 (b) 2, 3 and 4 (c) 1 and 4 (d) All of these

115. What does low-carbon growth imply in the present context?


1. More emphasis on the use of renewable sources of energy.
2. Less emphasis on manufacturing sector and more emphasis on agricultural sector.
3. Switching over from monoculture practices to mixed farming.
4. Less demand for goods and services.
Select the correct answer using the codes given below
(a) Only 1 (b) 2, 3 and 4 (c) 1 and 4 (d) None of these

Head Office: 127, Zone II, MP Nagar, Bhopal |+91-9111555433|www.legaledge.in Page 29 of 35


MOCK CLAT #04

116. Which of the following conditions is/are necessary for sustainable economic growth?
1. Spreading of economic prosperity more.
2. Popularising/spreading of adaptive technologies widely.
3. Investing on research in adaptation and mitigation technologies.
Select the correct answer using the codes given below
(a)Only 1 (b) 2 and 3 (c) 1 and 3 (d) All of these

117. Which of the following inferences can be made from the passage?
1. Rain fed crops should not be cultivated in irrigated areas.
2. Farming under water-deficient areas should not be a part of development strategy.
Select the correct answer using the codes given below
(a) Only 2 (b) Both land 2
(c) Neither 1 nor 2 (d) None of the above

118. Consider the following assumptions


1. Sustainable economic growth demands the use of creative potential of men.
2. Intensive agriculture can lead to ecological backlash.
3. Spread of economic prosperity can adversely affect the ecology and environment.
With reference to the passage, which of the above assumption (s) is/are valid?
(a) Only 1 (b) 2 and 3 (c) 1 and 3 (d) All of these

119. Which one of the following statements constitutes the central theme of the passage?
(a) Countries with greater economic prosperity are better equipped to deal with the consequences of
climate change
(b) Adaptation and mitigation should be integrated with development strategies
(c) Rapid economic growth should not be pursued by both developed and developing economies
(d) Some countries resort to overexploitation of natural resources for the sake of rapid development

Passage (Q.120–Q.124):
Peter Parker: To be honest, I feel that airplanes are much safer. When we are travelling by an airplane,
we face almost no hurdles in the sky, if not no hurdles at all; like there‘s no traffic or any object in the way
which the plane can collide with. Also, we have to consider that the auto-pilot technology employed in a
plane gives airplanes a huge advantage than rail or road transport. The less we depend on humans the
lesser is the chance of accidents.

Bruce Wayne: Okay, so the plane flies in the skies, right? You say there are no hindrances in the way.
Well, there have been so many instances of birds entering into the engines of the airplane. If such cases
happen, then remember you will be falling from such a height that you will crash into smithereens. In rail
travel or road travel, there is at least no chance of that.

120. Which of the following resembles a lot with the pattern of reasoning as has been employed by Peter
Parker?
(a) If we use robots for surgery, then we will have much higher success rates in surgery
(b) The use of smart-phones has made people smarter
(c) The more we rely on Artificial Intelligence, the more we get dumber
(d) Both a and b

121. Which of the following statements if true will weaken the arguments of Peter Parker?
(a) The pilots are highly-trained in manoeuvring the aircraft
(b) Sometimes there are air-pockets in the sky which can cause turbulence in air
(c) India‘s airlines use less advance and outdated technology
(d) The food served in airlines has deteriorated in the past few years

Head Office: 127, Zone II, MP Nagar, Bhopal |+91-9111555433|www.legaledge.in Page 30 of 35


MOCK CLAT #04

122. Which of the following can be the conclusion of Bruce Wayne‘s arguments?
(a) Air travel is safer than road or rail travel
(b) Air travel is definitely not safer than air or road travel
(c) We can‘t say that air travel is safer than road or rail travel
(d) None of these

123. Which of the following will strengthen Bruce Wayne‘s argument?


(a) Statistically there have been less air accidents than road or rail accidents
(b) There have been rising cases of drunk-driving
(c) Both a and b
(d) Neither a nor b

124. Which of the following roles does Peter Parker‘s statement ―The less we depend on humans the lesser is
the chance of accidents‖ play?
(a) It is the conclusion of his argument
(b) It is the argument for the conclusion
(c) It is the premise that Peter Parker has taken
(d) None of the above

Passage (Q.125–Q.129): Almost a century ago, a fad for sleep-learning swept the industrialised world,
ending only after neuroscientists determined it was physiologically impossible. Yet today, a growing body
of research suggests they were wrong. Sleep-learning appears to be heading for a revival, on a far more
solid scientific basis than its earlier incarnation. By subjecting sleep to a few engineering fixes, we could
minimise the time our brains are offline each night, gaining precious hours for absorbing information.
Over many nights, we could vastly expand our stock of knowledge and skills, or even treat stubborn
addictions and psychological traumas. All of which raises an unsettling question: should the prospect be
welcomed or dreaded? If we harness sleep for self-improvement, will we lose something essential about
ourselves?
The first person to make money from the concept was Alois Benjamin Saliger, a Czech-born New York-
based businessman and inventor –who in 1932 patented the Psycho-Phone. A phonograph fitted with a
repeating mechanism and a tiny acoustic horn, the device was meant to sit by a sleeper‘s bed and replay
spoken-word recordings at the volume of a whisper. It was marketed with disks whose titles included
Prosperity, Inspiration, Normal Weight, and Mating. ‗I desire an ideal mate,‘ Saliger intoned on the latter
record. ‗I radiate love. I have a fascinating and attractive personality. I have a strong sex appeal.‘
The Psycho-Phone operated on the premise of an unproven theory picked up by Aldous Huxley in his
dystopian novel, Brave New World (1932). There, recorded messages are used to train sleeping children
in the values of a soulless future society. A proud official in the book calls the new method, dubbed
‗hypnopaedia‘ by Huxley, the ‗greatest moralising and socialising force of all time‘.
The backlash came in the 1950s, when scientists began using electroencephalography (EEG), in which
electrodes are used to measure the brain‘s electrical activity. With this technique, they could finally
determine whether subjects were, in fact, asleep rather than drifting near sleep or just resting. When the
Rand Corporation researchers William Emmons and Charles Simon repeatedly played a list of 10 words
to men whose EEGs showed an absence of alpha waves (a reliable gauge of sleep), their performance
on a memory test upon awakening was no better than chance. Other EEG-monitored trials drew similar
results. Scientific consensus soon concluded that the sleeping brain was incapable of absorbing outside
information, and hypnopaedia was consigned to the realm of quackery.

125. Which of the following can be inferred from the passage?


(a) Neurologists believe that sleep-learning is physiologically impossible.
(b) Only by making quick engineering fixes can we make sleep learning possible.
(c) Both 1 and 2
(d) Neither 1 nor 2

Head Office: 127, Zone II, MP Nagar, Bhopal |+91-9111555433|www.legaledge.in Page 31 of 35


MOCK CLAT #04

126. Which of the following is the flaw in author‘s argument about sleep learning being a possibility today?
(a) He believes that human brain can be subjected to engineering fixes.
(b) He equated learning to absorbing information.
(c) He fails to take into account the fact that sleep is a time for digesting data, not ingesting it.
(d) None of the above

127. Which of the following, if true, will strengthen the belief in the theory about hypnopaedia?
(a) Scientific studies seemed to show it worked. The technique grew particularly popular in the Soviet
Union, where whole villages were said to learn foreign languages while dozing
(b) In one study, a group of sleeping men heard a recorded list of Chinese words and their English
translations; the next day, they scored significantly better on a comprehension test than a control
group.
(c) In a study, 20 boys with a nail-biting habit were played the phrase ‗my fingernails taste terribly bitter‘
300 times a night for 54 nights; by the end of the trial, 40 per cent had reportedly overcome their vice.
(d) All of the above

128. Which of the following could be a possible application of hypnopaedia?


(a) Mass indoctrination
(b) A tool for teaching new skills or changing unwanted
(c) Both 1 and 2
(d) Neither 1 nor 2

129. On what premise did the psycho phone operate?


(a) If the machine functioned as advertised, the user would awaken filled with irresistible confidence,
ready to stride off and conquer his chosen territory.
(b) People are as suggestible while asleep as they are under hypnosis.
(c) People can learn while they‘re asleep
(d) All of the above.

Directions (Q.130 –Q.133): Read the text and the statements carefully and answer the questions.

Four people who are expert in different languages live on the same side of a street in four houses each of
different colour. Each person also has a different favourite drink. The following additional information is
also known
A. The expert in English lives in the red house.
B. The expert in French drinks green tea.
C. The expert in Hindi lives in the first house on the left.
D. In the second house from the right they drink milk.
E. The Hindi expert lives adjacent to the blue house.
F. The Spanish expert drinks juice.
G. Green tea is drunk in the blue house.
H. The white house is to the right of the red house.
I. Coca-Cola is drunk in the yellow house.

130. Milk is drunk by :


(a) Hindi expert
(b) Englishman
(c) French expert
(d) None of them

Head Office: 127, Zone II, MP Nagar, Bhopal |+91-9111555433|www.legaledge.in Page 32 of 35


MOCK CLAT #04

131. The Hindi expert drinks:


(a) milk
(b) coco-cola
(c) green tea
(d) fruit juice

132. The color of the Hindi expert‘s house is:


(a) yellow
(b) white
(c) blue
(d) red

133. Which of the following is not true?


(a) Milk is drunk in the red house
(b) French expert lives in the blue house
(c) The Spanish expert lives in a corner house
(d) The French expert lives next to the Spanish expert

134. Complete the number series given below?


13 19 37 ? 141 243
(a) 73
(b) 67
(c) 75
(d) 65

135. What will come in the place of the (?) in the following number series?
5, 14, 45, 130, ?
(a) 234
(b) 397
(c) 405
(d) 417

Directions (Q.136): The following question is followed by information given in three statements. You
have to study the question along with the statements and decide the information given in which
statement(s) is necessary and sufficient to answer the question.

136. How is M related to R?


I. T, the only son of M, has two sisters.
II. M‘s son is the brother of the only sister of R.
III. R and T are children of M.
(a) Only I and III
(b) Only II
(c) Either I or II
(d) None of these

Head Office: 127, Zone II, MP Nagar, Bhopal |+91-9111555433|www.legaledge.in Page 33 of 35


MOCK CLAT #04

SECTION - E :QUANTITATIVE TECHNIQUES

Direction: (Q.137- Q.141). Study the following graph carefully and answer the following questions given
below.
Graph 1 : Boys + Girls
Graph 2 : Boys - Girls
Graph-1

Graph-2

137. What is the difference between Number of Girls in School A and Number of Girls in School B?
(a) 100 (b) 101 (c) 102 (d) 103

138. Girls in School C forms approximately what percent of the total number students in that School?
(a) 45.5% (b) 47.5% (c) 48.5% (d) 49.5%

139. What is the ratio of Sum of Boys in School D and Girls in School E together to the Sum of Girls in School
D and Boys in School E together is?
(a) 997:1012 (b) 999:1012 (c) 1000:1011 (d) 1000:1013

140. How many number of Boys are there in School F?


(a) 5134 (b) 5234 (c) 5334 (d) 5444

Head Office: 127, Zone II, MP Nagar, Bhopal |+91-9111555433|www.legaledge.in Page 34 of 35


MOCK CLAT #04

141. In which of the following School least no of Girls are present?


(a) School B (b) School C (c) School D (d) School E

Direction (Q.142–Q.146) : In a science exhibition 2000 student took par. 25%of the total are participating
in Raman House.50% of the remaining are participating Darwin and Dalton house in the ratio of the 3:2
and 20% of the rest are participating all houses. Rest are Raman and Darwin, Darwin and Dalton house
and Raman and Dalton house in the ratio of the 7:8:10.

142. The %of the student who did not participating in any house?
(a) 60 % (b) 10 % (c) 0 % (d) 20 %

143. What the %of the students who participate in only one house?
(a) 62.5% (b) 25 % (c) 50 % (d) None of these

144. What is the ratio of the number of student who participate in only Dalton house those who participate in
only Raman and Dalton House?
(a) 3 : 40 (b) 3 : 5 (c) 5 : 4 (d) 1:5

145. Find the number of students who participate in only one house?
(a) 500 (b) 1250 (c) 800 (d) 150

146. The students who participate in all three houses is what %of the student who participate in two house?
(a) 100% (b) 50% (c) 150 % (d) 25 %

Direction (Q.147- Q.150): Given table shows the quantity of Rice and tea (in metric tons) exported from
different countries in 2017 and quantity imported (in percentage) with respect to last year import of six
country in 2017.

Rice Tea
Nations
Export Import Export Import
India 5000 120% 240 40%
South Africa 4500 110% 140 70%
China 3760 80% 220 40%
U.S. 3800 100% 60 140%
Brazil 4100 90% 110 125%
Japan 2600 90% 135 100%

147. India's imported rice and imported tea are in the ratio of 2 :1 in year 2017. If in 2016 rice imported by
India is 120 metric ton then find the sum of total export (rice and tea) of India in 2017 and total import rice
and tea) of India in 2016.
(a) 5440 (b) 5520 (c) 5540 (d) 5515

148. If total import of Japan in 2016 is 30% of what it exported in 2017 and ratio between rice to tea imported
in 2017 is 360: 347 then what amount of rice is imported by Japan in 2017?
(a) 540 (b) 520 (c) 480 (d) 460

149. Rice imports of all countries are same in 2016 and Tea imports of all countries are same in 2017. If Rice
and Tea import of China in 2016 are in the ratio of 5 : 7 then find the ratio total Rice import in 2017 to tea
import in 2016 by countries together?
(a) 313: 450' (b) 451: 850 (c) 425 : 313 (d) 41 : 50

150. Total export by U.S. in 2017 is what percent less/more than the total export of Brazil in 2017
(approximately)?
(a) 7% (b) 9% (c) 4% (d) 8%

Head Office: 127, Zone II, MP Nagar, Bhopal |+91-9111555433|www.legaledge.in Page 35 of 35

You might also like